Stay ahead of learning milestones! Enroll in a class over the summer!

G
Topic
First Poster
Last Poster
No topics here!
Mount Inequality erupts on a sequence :o
GrantStar   87
N Mar 11, 2025 by Ilikeminecraft
Source: 2023 IMO P4
Let $x_1,x_2,\dots,x_{2023}$ be pairwise different positive real numbers such that
\[a_n=\sqrt{(x_1+x_2+\dots+x_n)\left(\frac{1}{x_1}+\frac{1}{x_2}+\dots+\frac{1}{x_n}\right)}\]is an integer for every $n=1,2,\dots,2023.$ Prove that $a_{2023} \geqslant 3034.$
87 replies
GrantStar
Jul 9, 2023
Ilikeminecraft
Mar 11, 2025
Mount Inequality erupts on a sequence :o
G H J
G H BBookmark kLocked kLocked NReply
Source: 2023 IMO P4
The post below has been deleted. Click to close.
This post has been deleted. Click here to see post.
GrantStar
815 posts
#1 • 23 Y
Y by EthanWYX2009, Amir Hossein, OronSH, mathfan2020, GeoKing, Supercali, Mogmog8, mathmax12, dangerousliri, oolite, Schur-Schwartz, centslordm, ismayilzadei1387, ihatemath123, Lamboreghini, Sedro, buddyram, aidan0626, megarnie, seifbaba, A21, cubres, farhad.fritl
Let $x_1,x_2,\dots,x_{2023}$ be pairwise different positive real numbers such that
\[a_n=\sqrt{(x_1+x_2+\dots+x_n)\left(\frac{1}{x_1}+\frac{1}{x_2}+\dots+\frac{1}{x_n}\right)}\]is an integer for every $n=1,2,\dots,2023.$ Prove that $a_{2023} \geqslant 3034.$
This post has been edited 6 times. Last edited by GrantStar, Jul 9, 2023, 4:54 AM
Reason: Renaming source
Z K Y
The post below has been deleted. Click to close.
This post has been deleted. Click here to see post.
beansenthusiast505
26 posts
#2 • 1 Y
Y by cubres
inequality has returned, my prayers were not answered (i’m not in imo and prob trying later but it should be p1/4 difficulty)
This post has been edited 1 time. Last edited by beansenthusiast505, Jul 9, 2023, 4:45 AM
Z K Y
The post below has been deleted. Click to close.
This post has been deleted. Click here to see post.
qwedsazxc
167 posts
#3 • 13 Y
Y by mathleticguyyy, adorefunctionalequation, Hyperabola, Geometry285, GeoKing, Rebel_1, rightways, Celestialer, centslordm, Lamboreghini, aidan0626, Lichenw, cubres
Let's show that $a_{n+2}\geq a_n+3$ for all $n$. By C-S and AM-GM:
$$a_{n+2}\geq a_n+\sqrt{\frac{(x_{n+1}+x_{n+2})^2}{x_{n+1}x_{n+2}}}>a_n+2.$$Therefore $a_{n+2}\geq a_n+3$. Since $a_1=1$, $a_{2023}\geq a_{2021}+3\geq\cdots\geq a_1+3033=3034$.
Z K Y
The post below has been deleted. Click to close.
This post has been deleted. Click here to see post.
EthanWYX2009
842 posts
#4 • 7 Y
Y by GeoKing, ike.chen, ys-lg, Euler_Gauss, ehuseyinyigit, Lamboreghini, DrPoe1898
Motivation. It is obvious to see that $3034=2022\times\frac 32+1,$ which leads us to the lemma below$.$
Lemma. For $\forall n\in\mathbb Z_+,a_{n+2}\geq a_n+3.$
Lemma Proof. As $a_n,a_{n+2}\in\mathbb Z_+,$ we only need to prove that $a_{n+2}>a_n+2.$ Using Cauchy inequality$,$
$$\begin{aligned}a_{n+2}=\sqrt{\sum\limits_{k=1}^{n+2}x_k\sum\limits_{k=1}^{n+2}\frac 1{x_k}}&\geqslant\sqrt{\sum\limits_{k=1}^{n}x_k\sum\limits_{k=1}^{n}\frac 1{x_k}}+\sqrt{(x_{n+1}+x_{n+2})\left(\frac 1{x_{n+1}}+\frac 1{x_{n+2}}\right)}\\&=a_n+\sqrt{4+\frac{(x_{n+1}-x_{n+2})^2}{x_{n+1}x_{n+2}}}>a_n+2.\blacksquare\\\end{aligned}$$Proof. Using Lemma we have $a_{2023}\geq a_{2021}+3\geq\cdots\geq a_1+3\times 1011=3034.\blacksquare$
This post has been edited 2 times. Last edited by EthanWYX2009, Jul 9, 2023, 1:30 PM
Z K Y
The post below has been deleted. Click to close.
This post has been deleted. Click here to see post.
juckter
322 posts
#5 • 2 Y
Y by Johnson100, Saniva_Rakib_Soha
We claim $a_{n + 2} \ge a_n + 3$ which is clearly enough. Let $s_n = x_1 + x_2 + \dots + x_n$ and $h_n = \frac{1}{x_1} + \frac{1}{x_2} + \dots + \frac{1}{x_n}$. Then

\[a_{n + 1}^2 = s_{n + 1}h_{n + 1} = (s_n + x_{n + 1})\left(h_n + \frac{1}{x_{n + 1}}\right) = s_nh_n + \left(\frac{s_n}{x_{n + 1}} + x_nh_{n + 1}\right) + 1 = a_n^2 + \left(\frac{s_n}{x_{n + 1}} + x_{n + 1}h_n\right) + 1\]
By AM-GM we have $\frac{s_n}{x_{n+1}} + x_{n+1}h_n \ge 2\sqrt{s_nh_n} = 2a_1$ and therefore $a_{n + 1}^2 \ge a_n^2 + 2a_n + 1 = (a_n + 1)^2$. Thus $a_{n + 1} \ge a_n + 1$ for all $n$, with equality if and only if $\frac{s_n}{x_{n+1}} = x_{n+1}h_n$, or $x_{n+1}^2 = \frac{s_n}{h_n}$. We finally prove that equality cannot happen twice in a row. Otherwise we have

\[x_{n + 1}^2 = \frac{s_{n + 1}}{h_{n + 1}} = \frac{s_n + x_{n + 1}}{h_n + \frac{1}{x_{n + 1}}} = \frac{s_n + \sqrt{\frac{s_n}{h_n}}}{h_n + \sqrt{\frac{h_n}{s_n}}} = \frac{s_n}{h_n} = x_n^2\]
A contradiction as $x_1, x_2, \dots, x_{2023}$ are all distinct.
Z K Y
The post below has been deleted. Click to close.
This post has been deleted. Click here to see post.
S.Ragnork1729
214 posts
#6 • 1 Y
Y by Johnson100
Solution : We will move on with a claim first.
Claim : $a_{n+2} > a_n+2$
Proof : Observe that $$a_{n+2} > a_n+2$$$$ \Longleftrightarrow \sqrt{(x_1+....+x_{n+2})(\frac{1}{x_1}+....+\frac{1}{x_{n+2}}} > \sqrt{(x_1+....+x_{n})(\frac{1}{x_1}+....+\frac{1}{x_{n}}} +2 .$$Now let $y=\sum_{i=1}^{n} x_i$ and $z=\sum_{i=1}^{n} \frac{1}{x_i}$ , then we have to show $$\sqrt{(y+x_{n+1}+x_{n+2})(z+\frac{1}{x_{n+1}}+\frac{1}{x_{n+2}})} > \sqrt{yz}+2$$$$  \Longleftrightarrow yz+y(\frac{1}{x_{n+1}}+\frac{1}{x_{n+2}})+z(x_{n+1}+x_{n+2})+(x_{n+1}+x_{n+2})(\frac{1}{x_{n+1}}+\frac{1}{x_{n+2}}) > yz+4\sqrt{yz}+4$$and it is obvious by AM-GM.
Hence we get $$a_{2023} \geq a_{2021}+3 \geq a_{2019}+6 \geq \cdots a_4+3030 \geq a_1+3033 =1+3033=3034 .$$Note : The problem was very good but the main part was to prove $a_{n+2} \geq a_{n}+3$ using AM-GM and Algebraic Expansion.
This post has been edited 2 times. Last edited by S.Ragnork1729, Jul 9, 2023, 6:08 AM
Z K Y
The post below has been deleted. Click to close.
This post has been deleted. Click here to see post.
mathuz
1512 posts
#7
Y by
Obviously, $a_n$ should be an increasing (strictly) sequence of positive integers with $a_1=1$. We will show that $a_{n+1}=a_n+1$ and $a_{n+2} = a_{n+1}+1$ doesn't hold at the same time. Assume it does.

We know that \[ a_{n+1}^2-a_n^2=x_{n+1}\left(\frac{1}{x_1}+\ldots +\frac{1}{x_{n}}\right) + \frac{1}{x_{n+1}} \left(x_1+\ldots +x_n\right)+1 = 2a_n+1 \]and \[ 2a_n = x_{n+1}\left(\frac{1}{x_1}+\ldots +\frac{1}{x_{n}}\right) + \frac{1}{x_{n+1}} \left(x_1+\ldots +x_n\right) \ge 2a_n \]by AM-GM, so we should have \[ x_{n+1}\left(\frac{1}{x_1}+\ldots +\frac{1}{x_{n}}\right) = \frac{1}{x_{n+1}} \left(x_1+\ldots +x_n\right) = a_n. \]Similarly, we can obtain \[ x_{n+2}\left(\frac{1}{x_1}+\ldots +\frac{1}{x_{n+1}}\right) = \frac{1}{x_{n+2}} \left(x_1+\ldots +x_{n+1}\right) = a_{n+1}. \]That means \[ a_nx_{n+1}+x_{n+1}=x_1+x_2+\ldots +x_n+x_{n+1} = a_{n+1}x_{n+2} \]and since $a_{n+1}=a_n+1$ we get that $x_{n+1}=x_{n+2}$, which is a contradiction.

Hence, if $a_{n+1}-a_n=1$ then $a_{n+2}-a_{n+1}>1$, or vice versa. In any case, $a_{n+2}-a_n\ge 3$ and $a_{2023} \ge a_1+3\cdot 1011 = 3034$.
Z K Y
The post below has been deleted. Click to close.
This post has been deleted. Click here to see post.
GrantStar
815 posts
#8 • 8 Y
Y by RobertRogo, GeoKing, centslordm, ehuseyinyigit, Lamboreghini, channing421, aidan0626, Jack_w
Solution without words

Reason that doing the same thing for gaps of one fails
This post has been edited 2 times. Last edited by GrantStar, Jul 9, 2023, 6:12 AM
Z K Y
The post below has been deleted. Click to close.
This post has been deleted. Click here to see post.
Alfredfeed
9 posts
#9
Y by
Solution without using AM-GM
Lets prove $a_{n+1}$$\geq$$a_{n-1}+3$

s = $x_1 + x_2 + ... + x_{n-1}$

h = $\frac{1}{x_1} + \frac{1}{x_2} + ... + \frac{1}{x_{n-1}}$

$a^2_{n-1}$ = hs

$a^2_{n} = (s+x_n)(h+\frac{1}{x_n})$

$a^2_{n} = hs + \frac{s}{x_n} + {x_n}h + 1$

$a^2_{n-1} = a_{n-1}^2 + \frac{s}{x_n} + xh + 1$

assume $a^2_{n}$=$a_{n-1}+1$;$a^2_{n+1}$=$a_{n}+1$

otherwise its trivial since obviously $a_{n-1}<a_{n}$

2$a_{n-1}=\frac{s}{x_n}+xh$

$h=\frac{a^2}{s}$

2$a_{n-1}=\frac{s}x+\frac{x}s{a_{n-1}^2}$

let $k = \frac{s}x$

2$a_{n-1}$=k+$\frac{1}k{a_{n-1}^2}$

solving for $a_{n-1}$, we get:

$a_{n-1}$=$\frac{s}x$

s=$a_{n-1}$x

$a_{n-1}+1=\frac{s+x}x'$

x'(a+1)=x(a+1)

x'=x
contradiction, so $a_{n+1}$$\geq$$a_{n-1}+3$
Z K Y
The post below has been deleted. Click to close.
This post has been deleted. Click here to see post.
AnonymousBunny
339 posts
#10 • 1 Y
Y by GeoKing
The crucial claim is the following:

Claim: For any $n$, it cannot be the case that $a_{n+1} = a_n + 1$ and $a_{n+2} = a_{n+1}+1$.
Proof:
Suppose, FTSOC, $a_{n+1} = a_n + 1, a_{n+2} = a_{n+1}+1$.

$$ a_{n+1} = a_n + 1 \iff x_{n+1} \left( x_1 + x_2 + \cdots + x_n \right) + \dfrac{1}{x_{n+1}} \left(x_1 + \cdots + x_n \right) = 2a_n$$

Treat this as a quadratic in $x_{n+1}$: its discriminant vanishes and the only root is $x_{n+1} = \dfrac{a_n}{1/x_1 + \cdots + 1/x_n}$.
Similarly, $x_{n+2} = \dfrac{a_{n}+1}{1/x_1 + \cdots + 1/x_{n+1}}$. I now claim that $x_{n+1} = x_{n+2}$, which is a contradiction. This follows from a simple calculation:


$$ x_{n+1} = x_{n+2} \iff a_n \left( \dfrac{1}{x_1} + \cdots + \dfrac{1}{x_{n+1}}\right) = (a_n+1) \left( \dfrac{1}{x_1} + \cdots + \dfrac{1}{x_n} \right) \iff x_{n+1} = \dfrac{a_n}{1/x_1+\cdots + 1/x_n} \blacksquare$$
Now, note that $a_2 \geq 2$ by Cauchy-Schwartz. Since equality cannot hold (as $x_1, x_2$ are distinct), in fact, $a_2 \geq 3$. Also note that $a_n$ is strictly increasing. The claim shows that $a_i$ must jump more than one once every two steps, i.e., $a_{n+2} \geq a_n + 3$. Applying this inequality repeatedly gives the result.
Z K Y
The post below has been deleted. Click to close.
This post has been deleted. Click here to see post.
Manteca
52 posts
#11 • 2 Y
Y by ZNatox, GeoKing
$$a_{n+1}^2 = a_n^2 + 1 + x_{n+1}(\frac{1}{x_1}+ \cdots + \frac{1}{x_{n}}) + \frac{1}{x_{n+1}}(x_1 + \cdots + x_{n}) \geq a_n^2 + 1 + 2a_n = (a_n+1)^2$$By AM-GM, with equality if and only if the two things we are summing are equal, in particular $x_{n+1} = \sqrt{\frac{A}{B}}$ where $A = \sum a_i$ and $B = \sum \frac{1}{a_i}$.
If also $a_{n+2} = a_{n+1} + 1$ then again we must have $x_{n+2} = \sqrt{\frac{A+\sqrt{\frac{A}{B}}}{B+\sqrt{\frac{B}{A}}}}$ but both things inside the square root are equal so $x_{n+1} = x_{n+2}$ but that cant happen so at least $a_{n+2} \geq a_n + 3$. Finally $a_{2023} \geq 1 + 3 \cdot 1011 = 3034$ as we wanted to show.
This post has been edited 1 time. Last edited by Manteca, Jul 9, 2023, 6:37 AM
Z K Y
The post below has been deleted. Click to close.
This post has been deleted. Click here to see post.
djmathman
7936 posts
#12 • 2 Y
Y by Assassino9931, RobertRogo
Sneaky little bugger.

Define $A_n := x_1 + x_2 + \cdots + x_n$ and $B_n := \tfrac 1{x_1} + \tfrac1{x_2} + \cdots + \tfrac1{x_n}$. Observe that
\begin{align*}
a_{n+1}^2 &= \left(A_n + x_{n+1}\right)\left(B_n + \frac{1}{x_{n+1}}\right) \\
&= A_nB_n + \frac{A_n}{x_{n+1}} + B_n x_{n+1} + 1 \\
&\geq A_nB_n + 2\sqrt{A_nB_n} + 1 = (a_n + 1)^2.
\end{align*}Hence $a_{n+1} \geq a_n + 1$.

We now prove the stronger inequality $a_{n+2} \geq a_n + 3$. Suppose, by way of contradiction, that $a_{n+2} = a_n + 2$. Then equality cases in both inequalities above (i.e. for $a_{n+1}$ and $a_{n+2}$) hold, so $x_{n+1} = \sqrt{\tfrac{A_n}{B_n}}$ and
\[
x_{n+2}^2 =\frac{A_n + x_{n+1}}{B_n + \frac 1{x_{n+1}}} = \frac{A_n + \sqrt{\frac{A_n}{B_n}}}{B_n + \sqrt{\frac{B_n}{A_n}}}= \frac{A_n}{B_n}.
\](The last equality is true because $\tfrac{x_{n+1}}{\frac{1}{x_{n+1}}} = x_{n+1}^2 = \tfrac{A_n}{B_n}$.) This contradicts the fact that the $x_j$ are pairwise distinct.

Remark. I think the claims here are motivated, but the main sticking point for me was getting over my confusion about why the bound wasn't tighter. (It didn't help that I made a few algebra mistakes along the way.) Proving the inequality $a_{n+2} \geq a_n + 3$ directly is certainly faster, but this solution shows why we need to consider this stronger bound in the first place.
Z K Y
The post below has been deleted. Click to close.
This post has been deleted. Click here to see post.
Minkowsi47
48 posts
#13
Y by
MOTIVATION : The thing which motivate $a_{n+2} \geq a_{n}+3$ is $a_{2023}\geq 3034$ as actually this is the point for me which makes me realise to follow up this claim. proving that claim is same as GrantStar did and finishes up the problem. :-D
This post has been edited 1 time. Last edited by Minkowsi47, Jul 9, 2023, 7:35 AM
Z K Y
The post below has been deleted. Click to close.
This post has been deleted. Click here to see post.
Tintarn
9031 posts
#14 • 2 Y
Y by carefully, ehuseyinyigit
So. Is the bound $3034$ actually sharp?
Z K Y
The post below has been deleted. Click to close.
This post has been deleted. Click here to see post.
PythZhou
6 posts
#15
Y by
Too easy for IMO inequality.
Claim : $a_{n+2}>a_{n}+2$ which is equivalent to $a_{n+2} \ge a_{n}+3$ since $a_n$ is an integer. Hence $$a_{2023} \ge a_1+\sum_{n=1}^{1011}3=3034$$
Then prove the claim:

We define $X_n=x_1 + x_2 + \cdots + x_n$ and $Y_n=\frac{1}{x_1}+\cdots+\frac{1}{x_{n}}$, so $a_n=\sqrt{X_n Y_n}$
\begin{align*}
a_{n+2}=&\sqrt{(x_1 + x_2 + \cdots + x_n+x_{n+1}+x_{n+2}) (\frac{1}{x_1}+\cdots+\frac{1}{x_{n}}+\frac{1}{x_{n+1}}+\frac{1}{x_{n+2}})} \\
&= \sqrt{(X_n+x_{n+1}+x_{n+2})(Y_n+\frac{1}{x_{n+1}}+\frac{1}{x_{n+2}})} \\
&= \sqrt{a_n^2+X_n(\frac{1}{x_{n+1}}+\frac{1}{x_{n+2}})+Y_n(x_{n+1}+x_{n+2})+(x_{n+1}+x_{n+2})(\frac{1}{x_{n+1}}+\frac{1}{x_{n+2}})} \\
&> \sqrt{a_n^2+ 4 \sqrt{X_nY_n}+4} \\
&= \sqrt{a_n^2+4a_n+4}=a_n+2
\end{align*}
Z K Y
G
H
=
a